Set 2 Comprehensive PA

Réussis tes devoirs et examens dès maintenant avec Quizwiz!

Which of the following inhibits bone resorption and can be used for the treatment of Paget disease in pts who are symptomatic? a. adalimumab b. alendronate c. anakinra d. probenecid e. sulfasalazine

(B) Bisphosphonates, such as alendronate, inhibit bone resorption with relatively few side effects. They have been widely used for the prevention and treatment of postmenopausal osteoporosis, but are also primary agents used in the initial treatment of Paget disease. (Seton, 2013)

Which of the following antineoplastic meds is most likely to cause cardiac toxicity and precipitate heart failure? a. cisplatin b. cyclophosphamide c. doxorubicin d.. tamoxifen e. 6-Mercaptopurine

(C) Doxorubicin is a common antineoplastic drug used for a variety of cancers, including breast, bladder, ovarian, and endometrial, among many others. It has well-established, dose-dependent cardiotoxic effects.

A 28 yr old female in the ED is administered an IV paralytic agent prior to ET intubation. The agent produces transient muscle fasciculation's, particularly over the thorax and abdomen, prior to paralysis. Which of the following was the patient most likely administered? a. carbamazepine b. pyridostigmine c. rocuronium d. succinylcholine e. tubocurarine

(D) Succinylcholine Tubocurarine and rocuronium are classified as nondepolarizing neuromuscular blocking drugs, whereas succinylcholine is depolarizing. Nondepolarizing agents competitively block nicotinic receptors on skeletal muscle, which leads to flaccid muscle paralysis. Depolarizing agents, on the other hand, activate nicotinic receptors on skeletal muscle cells leading to membrane depolarization, initial fasciculations, and intense contractions. Succinylcholine is not metabolized efficiently at neuromuscular junctions; hence, the cells remain depolarized and are unable to repolarize or recover back to a resting state. This failure to repolarize then leads to a flaccid muscle paralysis. Pyridostigmine in an acetylcholinesterase inhibitor indicated for myasthenia gravis and causes an increase in skeletal muscle activity.

Hyperkalemia is a CI to the use of which of the following medications? a. amiloride b. cimetidine c. glipizide d. metformin e. verapimil

(A) Amiloride is a K+ -sparing diuretic based on its mechanism of action. In the kidneys, it will lead to less K+ excretion in the urine and hence retention of plasma K+ . In patients with elevated plasma K+ , amiloride can cause further hyperkalemia, which can impact neuromuscular and cardiac function

A 36-year-old woman with no past medical history presents to the emergency room with sudden onset of left sided chest pain and shortness of breath. She denies any trauma but states that she recently drove 24 hours from Florida to New York in her car. She is on oral contraceptive pills and she smokes a half pack of cigarettes a day. Her vital signs are as follows: Temp: 98.6, HR: 128 and regular, BP 130/88, RR: 26, 02 sat 92% on room air. Her physical examination is unremarkable. Which of the following is the most appropriate next step in the evaluation of this patient? a. Pulmonary angiography b. Venous Doppler of the lower extremities C.Ventilation perfusion scan d. Spiral CT scan of the chest with contrast e. D-dimer

24 Choice D (Spiral CT scan of the chest) is the correct answer. This is the classic presentation of pulmonary embolism. All of the choices can be used in suspected pulmonary embolism so this question tests your ability to tell the indications and contraindications for each test. Spiral CT scan of the chest with IV contrast is considered the best initial screening test for pulmonary embolism in a patient with moderate to high clinical suspicion. There is a high clinical suspicion because she is a smoker, she is on oral contraceptives (hypercoagulability) and she had a car ride greater than 4 hours (venous stasis). Choice A (Pulmonary angiography) is indicated in patients in patients with a high clinical suspicion in whom noninvasive testing such as Spiral CT scan and/or ventilation perfusion scan are either contraindicated and/or inconclusive. Pulmonary angiography is the gold standard (definitive diagnosis) but because it is invasive, it is usually reserved for high probability patients in whom noninvasive testing is inconclusive. If the stem asked for the gold standard, this would have been the answer. Be sure to always answer what the question is asking. Choice B (Venous Doppler of the lower extremities) can be done to look for the source of the embolus as 95% of pulmonary emboli originate from the lower extremity or pelvic deep venous thrombi, but in a case where pulmonary embolism is suspected, a negative Doppler does not exclude pulmonary embolism as there are other potential sources for thrombi formation. Choice C (Ventilation perfusion scan) is often indicated in patients with moderate to high risk in whom CT scan is inconclusive or contraindicated (ex. patients who cannot tolerate IV contrast at increased risk of kidney injury). Choice E (D dimer) is often used but it is only clinical useful if the D-dimer is negative in low risk patients. This patient is high risk (see choice D).

Which of the following when combined w/ chemotherapy can be effective in treating Philadelphia chromosome positive acute lymphoblastic leukemia in adults? a. adalimumab b. imatinib c. methotrexate d. raloxifene e. tamoxifen

A key component of Philadelphia chromosome positive acute lymphoblastic leukemia therapy is the use of a BCR-ABL tyrosine kinase inhibitor (TKI), which is directed against the protein product of the Philadelphia chromosome. The TKI must be combined with other therapy since remissions induced by TKI therapy alone are short-lived. The largest experience has been with imatinib combined with conventional acute lymphoblastic leukemia chemotherapy regimens

Constipation, abdominal distention, bloating, and flatulence are common adverse effects associated with which class of drugs? a. bile acid sequestrants b. fibrates c. PPIs d. statins e. triptans

A) Bile acid sequestrants (e.g., cholestyramine, colestipol, colesevelam) are used for lowering LDL-C. They bind bile acids in the intestinal lumen, thereby preventing them from carrying out their normal functions of emulsification and micelle formation. Emulsification is an important process for lipid digestion, while the formation of micelles is required for lipid absorption. These actions not only inhibit lipid digestion and absorption from the intestinal lumen, but they also deplete the hepatic pool of cholesterol as a result of increased bile acid synthesis. Normally, bile acids are recirculated (enterohepatic circulation) from the intestine and back to the liver for reincorporation into the bile. Resins cause the bile acids to be excreted with the feces, so the liver needs to continually synthesize new bile acids from endogenous cholesterol. Constipation, abdominal distention, bloating, and flatulence result from the increased lipid content of the stool, because lipids are not being absorbed across the intestinal wall. These adverse effects can often be managed by increasing fluid and fiber intake and also using stool softeners.

A 72-year-old male is admitted for status asthmaticus and is given ipratropium nebulizer therapy and oral corticosteroids. The nurse informs you the next morning that the patient is exhibiting signs of acute confusion in which the patient fluctuates between agitation and somnolence, with paranoid ideation. There are no visual hallucinations. Which of the following is the most likely diagnosis? a. dementia b. delirium c. Type Il bipolar disorder d. Wernicke's encephalopathy e. Vascular dementia

B. Delirium is an acute, abrupt transient confused state due to an identifiable cause in this vignette, anticholinergics can cause delirium in come patients as well as corticosteroids). There is usually full recovery within 1 week

10. A 45-year-old male with a history of diabetes mellitus and polycystic kidney disease is admitted to the intensive care unit for a subarachnoid hemorrhage. He is placed on stool softeners and phenytoin for seizure prophylaxis. His home medications are daily aspirin and chlorpropramide. He states over the last few days, he has developed nausea and fatigue. There are no signs of peripheral edema or decreased skin turgor. His vital signs are stable. Lab tests are as follows: PATIENT NORMAL Serum Glucose (mg/dL) 128 (64 - 128) Serum Sodium (mEq/L) 129 (135 - 145) Serum BUN (mg/dL) 7 (7 - 10) Serum Creatinine (mg/dL) 0.7 (0.8 - 1.4) Serum Osmolarity (mOsm/kg) 257 (280 - 290) Urine Osmolarity (mmol/kg) 680 (300 - 900) Urine sodium (mEq/L) 35 (20) Which of the following is considered the management of choice in this patient? a. hypertonic saline infusion with furosemide b. normal saline infusion c. fluid restriction d. fluid and sodium restriction e. demeclocycline administration

C (fluid restriction) is the correct answer. This patient has SIADH probably due to a combination of factors: the intracranial bleed, anticonvulsants such as phenytoin and chlorpropramide (a 1st generation sulfonylurea). The absence of peripheral edema or hypovolemia indicates he is isovolemic. The low serum osmolarity and the low sodium make it a hypotonic isovolemic hyponatremia. Hyponatremia is a problem of water overload not sodium. Therefore, the management of hypotonic isovolemic hyponatremia is fluid restriction and to treat the underlying causes. Choice A (hypertonic saline infusion with furosemide) is the management of severe hyponatremia (ex. serum sodium <120 mEq/L) or severe neurologic symptoms. Choice B (normal saline infusion) is indicated in hypovolemic hypotonic hyponatremia. In this vignette, there are no signs of hypovolemia: the creatinine is within normal limits, the BUN: creatinine ratio is less than 20:1 and there is no poor skin turgor. Choice D (fluid and sodium restriction) is indicated in hypervolemic hypotonic hyponatremia. There are no signs of edema in this patient. Choice E (demeclocycline administration) can be used in severe cases of SIADH.

A 12 year old boy presents to the pediatric clinic with facial swelling. He was treated three weeks ago for sinusitis and those symptoms have resolved. He is now complaining of unilateral eye edema with ocular discharge and decreased vision. There is pain with eye movement during the extra-ocular exam. There are no cells or flare seen nor is there any fluorescein uptake. Which of the following is the most likely diagnosis? a. bacterial conjunctivitis B. anterior uveitis C. Septal cellulitis D dacrocystitis E preseptal cellulitis

C. Septal cellulitis Orbital cellulitis is an infection of the cavity surrounding the eye. It is also associated w/ local superficial signs of infection. Due to the involvement of the ocular muscles it is assoc. w/ visual changes and pain with ocular movement. A. bacterial conjunctivitis is associated w/ conjunctival erythema and may have ocular discharge but since there is no ocular muscle involvement there should be no vision deficiits nor pain with ocular movements B. anterior uveitis assoc. w/ conjunctival erythema. TheHALLMARK of uveitis is the presence of cells and flare on examination D. Dacrocystitis is an inflammation of the lacrimal gland. it is not associated w/ visual changes or eye pain w/ ocular movement. It is associated w/ local signs of infection on the nasal side of the lower eyelid. E. Preseptal cellulitis presents very similar to septal cellulitis w/ periorbital pain and swelling. The distinguishing factor is the preseptal cellulitis is not associated w/ ocular pain or visual changes (that are seen in septal cellulitis since preseptal cellulitis does not involve the periorbital fat or the ocular muscles)

Which of the following is not a side effect of erythromycin? a. prolonged QT interval b. cytochrome P450 inhibition c. diarrhea d. flushing with rapid IV administration e. increased potential for muscle toxicity if used with HMG COA reductase inhibitors

CHOICE D (flushing with rapid IV administration) is the correct answer. Red man syndrome is histamine-induced flushing of the skin associated with rapid IV administration of vancomycin. Slowing down the infusion can reduce the flushing. All of the other choices are associated side effects of erythromycin.

A 23-year-old woman has a positive pregnancy test. She states that her last menstrual period was July 10 and that her periods are regular. Which of the following most accurately describes her estimated date of delivery according to Naegele's rule? a. April 17 b. April 3 c. October 17 d. October 10 e. May 17

Choice A (April 17) is the correct answer. Naegele's rule states that pregnancies last approximately 280 days (40 weeks) so to calculate it, use the first day of the menstrual period then subtract 3 months and add seven days to get a rough estimated due date.

A 40-year-old male who works in the Fulton Fish Market in the Bronx, NY comes into the corporate clinic for a rash on his hands. He states he may have sustained a puncture wound while packing shellfish. On physical examination of the hand, there is non-pitting edema, purplish erythema and sharp, irregular margins from the possible puncture site extending peripherally with central clearing. There are neither granulomas present nor crepitus. A radiograph of the affected hand shows no evidence of fracture, retained foreign body or gas in the soft tissues. A wound culture is positive for growth of gram-positive bacilli after 3 days. Which of the following organisms is the most likely cause? a. Erysipelothrix rhusiopathiae b. Mycobacterium marinum c. Coxiella burnetii d. Clostridium perfringens e. Haemophilus ducreyi

Choice A (Erysipelothrix rhusiopathiae) is the correct answer. Erysipeloid is an occupational disease that usually follows a skin abrasion or puncture wound from raw fish, shellfish, raw meat and poultry. It presents with a localized cutaneous lesion that is usually limited to the hand, finger and web spaces. It is associated with non pitting edema and purplish erythema with sharp irregular margins extending peripherally while clearing centrally Choice B (mycobacterium marinum) also known as "fish tank granuloma" is also associated with seafood handlers, marine workers and fisherman. It classically presents with an erythematous, bluish, raised papule or nodule at the site of trauma and may develop subsequent lesions along the path of lymphatic drainage. As a mycobacterium species, the cultures are slow growing and may often take weeks before growing out in cultures. There were no granulomas present in this vignette. Choice C (Coxiella burnetii) is the causative agent of Q fever. Choice D (Clostridium perfringens) causes gas gangrene. On physical examination, there are usually brownish bullae and crepitus on physical examination of the affected tissues (from the gas produced by C. perfringens) with the presence of gas in the tissues on X ray. Choice E (Haemophilus ducreyi) is the causative agent of chancroid, a sexually transmitted disease.

A 7-year-old girl presents with mild hearing loss and low-grade fever. On physical examination, there is erythema and an effusion of the tympanic membrane with decreased motility of the tympanic membrane on insufflation. There is absence of bullae on the tympanic membrane. Which of the following antibiotics are considered the first line management of choice? a. amoxicillin b. amoxicillin/clavulanic acid C. doxycycline d. ciprofloxacin e. metronidazole

Choice A (amoxicillin) is the correct choice. The erythematous tympanic membrane and decreased motility of the tympanic membrane are consistent with acute otitis media. Amoxicillin is considered the management of choice. Cephalosporins may also be used. Choice B (Amoxicillin/Clavulanic acid - Augmentin) is second line if not responsive to amoxicillin.

A 43-year-old female with no significant past medical history is complaining of 8 months of generalized fatigue and weakness. Her symptoms include cravings to eat ice and clay. For 3 months, she has been having progressive dysphagia. Physical examination reveals pallor of the conjunctiva, atrophic glossitis, angular cheilitis and spooning of the nails. An upper endoscopy is performed to evaluate the dysphagia, revealing the presence of esophageal webs. Which of the following is the most likely panel found in this patient? a. decreased serum iron, decreased ferritin, increased total iron binding capacity b. decreased serum iron, increased ferritin, decreased total iron binding capacity c. increased serum iron, normal ferritin, decreased total iron binding capacity d. normal serum iron, normal ferritin, decreased total iron binding capacity e. increased serum iron, increased ferritin, decreased total iron binding capacity

Choice A (decreased serum iron, decreased ferritin, increased total iron binding capacity) is the correct answer. The patient has Plummer-Vinson syndrome, which is characterized by the triad: dysphagia, esophageal webs, and atrophic glossitis in patients with iron deficiency anemia. Iron deficiency anemia is associated with a decreased serum iron, a decreased ferritin (ferritin is a protein that stores iron, in iron deficiency anemia the iron stores are used up). It is also associated with an increased total iron binding capacity. Total binding capacity evaluates the blood's ability to combine iron with its carrier molecule transferrin. In situations of low iron, more transferrin is produced, leading to a higher total iron binding capacity Choice B (decreased serum iron, increased ferritin, decreased total iron binding capacity) is associated with anemia of chronic disease. In anemia of chronic disease (due to chronic inflammation or infection), there is an increased ferritin because it is an acute phase reactant. This increased ferritin sequesters iron. The total iron binding capacity is decreased as there is less transferrin produced in anemia of chronic disease. Choice E (increased serum iron, increased ferritin, decreased total iron binding capacity) is seen with hereditary hemochromatosis. Hereditary hemochromatosis is excess iron deposition in the parenchymal cells of the heart, liver, pancreas and endocrine organs. This is associated with an increased serum iron (iron overload), increased transferrin saturation (transferrin is the iron carrying molecule, excess iron saturates the transferrin proteins) and a decreased total iron binding capacity (due to the higher serum iron levels). Lead poisoning may also present with similar lab findings. These disorders resemble anemia of chronic disease lab wise but the serum iron is elevated.

A 55-year-old male with a history of myocardial infarction treated with triple bypass presents to the clinic with symptoms that are consistent with chronic mesenteric ischemia. Which of the following would be expected? a. dull abdominal pain worse with meals and associated with weight loss b. left lower quadrant pain and tenderness with bloody diarrhea C. severe abdominal pain out of proportion to physical examination d. right upper quadrant pain e. flank pain that radiates to the testicle

Choice A (dull abdominal pain worse with meals and weight loss) is the correct answer. Chronic mesenteric ischemia is due to atherosclerosis of the arteries that supply the bowel. It can present with "intestinal angina" (abdominal pain worse with eating due to the increased demand of blood to the bowel in the setting of a decreased supply). Patients frequently develop anorexia due the fear of the abdominal pain associated with eating. Choice B (left lower quadrant pain and tenderness with bloody diarrhea) describes the pain of ischemic colitis. Choice C (severe abdominal pain out of proportion to physical examination) describes the pain of acute mesenteric ischemia. Choice D (diffuse abdominal pain relieved with defecation) describes the abdominal pain associated with irritable bowel syndrome. Choice E (flank pain that radiates to the testicle) describes the pain associated with nephrolithiasis.

A 54-year-old male presents with progressively worsening headaches. The headaches tends to be worse in the morning, waking the patient up at night. The patient also has some spatial disorientation. His vital signs are as follows: BP: 150/100, heart rate: 40 beats per minute, O2 sat: 99% on room air, Respirations: Cheyne Stokes breathing pattern. A CT scan shows a nonhomogeneous mass with a hypodense center, variable ring of enhancement with surrounding edema. The lesion crosses the corpus callosum giving it a "butterfly appearance". The lesion is not attached to the dura. Which of the following is the most likely diagnosis? a. glioblastoma multiforme b. oligodendroglioma c. ependymoma d. hemangioma e. meningioma

Choice A (glioblastoma multiforme) is the correct answer. Glioblastoma multiforme is the most common primary brain malignancy and classically shows up on CT scan as a nonhomogeneous mass with a hypodense center and variable ring of enhancement with surrounding edema with a butterfly appearance if it crosses the corpus callosum. Choice B (oligodendroglioma) can be found anywhere in the brain but it is not as common as glioblastoma multiforme. Choice C (ependymoma) often occurs in children and is most commonly seen in the 3rd or 4th ventricle. Choice D (hemangioma) is most commonly found in the cerebellum and the brainstem (though they may occur in the cerebral hemispheres and the spinal cord. They are associated with von-Hippel-Lindau syndrome (especially if there is retinal involvement). Choices E (meningioma) most commonly occurs in the convexities of the hemispheres and the parasagittal regions and are often attached to the dura.

A 32-year-old physician who recently moved to the United States from mainland China presents to the clinic for yearly tuberculosis screening. The patient is asked during routine history about any symptoms and he denies chest pain, hemoptysis, weight loss, fever or chills. A PPD is placed, and 48 hours later, reveals 10 mm of induration and 5 mm of erythema. A chest radiograph shows no acute cardiopulmonary disease or granuloma. Which of the following is considered the management of choice? a. isoniazid + pyridoxine (B6) total duration of treatment for 9 months b. isoniazid + rifampin + ethambutol + pyrazinamide (total treatment duration of 9 months) c. isoniazid + rifampin + ethambutol + streptomycin (total treatment duration of 6 months) d. isoniazid + pyridoxine (B6) total duration of treatment for 12 months e. ceftriaxone + azithromycin

Choice A (isoniazid + pyridoxine (B6) total duration of treatment for 9 months) is correct. This is a 2-step question. The first step is determining if the PPD is positive or not. This is a common thing asked on the boards. First you have to determine your cut off rate by his risk factors. 10 mm or greater is considered positive in both health care workers (he is a physician) and immigrants (he is from China). Now that you determine he is positive, the next step is to obtain a chest radiograph to rule out active tuberculosis. Once active tuberculosis is ruled out, the diagnosis is latent TB infection (meaning he is infected but not infectious as evident by a positive PPD, the absence of symptoms and a negative chest radiograph). You can offer the patient prophylaxis for latent TB infection. Treatment of LTBI reduces the incidence of secondary (reactive) TB in the future if the patient's immune system wanes (ex. getting older, HIV, steroid use, or chemotherapy). Treatment of latent TB infection is INH + B6 (to prevent peripheral neuropathy from the INH) for 9 months for the general population Choice B (isoniazid + rifampin + ethambutol + pyrazinamide total treatment duration of 9 months) is incorrect because 1) 4 drug treatment is used for active TB (which he does not have) and because active TB is usually treated for total 6 months duration. Choice C (isoniazid + rifampin + ethambutol + streptomycin (total treatment duration of 6 months)] is the treatment for active TB (positive PPD, active symptoms and positive X rays for infection) not latent TB. Choice D (isoniazid + pyridoxine (B6) total duration of treatment for 12 months) is the treatment of latent TB infection if the patient is HIV positive or has a granuloma seen on chest radiographs. Choice E (ceftriaxone + azithromycin) is the treatment for community acquired pneumonia treated as an inpatient.

A 21-year-old male is complaining of persistent swelling to the neck. He states the swelling has worsened over the last week when he celebrated his birthday and consumed large amounts of alcohol. On physical examination, there is nontender cervical lymphadenopathy. The swollen lymph node is excised. Which of the following biopsy findings are most consistent with Hodgkin's lymphoma? a. large B cells with bilobed or multilobar nuclei giving it an "owl eye appearance" due to eosinophilic inclusions in the nuclei. b. abnormal, fragile B-lymphocytes with a "smudged" appearance that occurs with slide preparation. C. crystalized granular elongated needles seen in the cytoplasm of immature white blood cells. d. red blood cell inclusions that are composed of denatured hemoglobin. e. reciprocal translocation between chromosome 9 and chromosome

Choice A (large B cells with bilobed or multilobar nuclei and an "owl eye appearance" due to eosinophilic inclusions in the nuclei) is the correct answer. This describes the Reed Sternberg cell, which is pathognomonic for Hodgkin lymphoma. Choice B (abnormal, fragile B lymphocytes with a "smudged" appearance that occurs with slide preparation) describes the smudge cells associated with chronic lymphocytic leukemia. Choice C (crystalized granular elongated needles seen in the cytoplasm of immature white blood cells) describes the Auer rod, which is associated with Acute myelogenous leukemia. Choice D (red blood cell inclusions that are composed of denatured hemoglobin) describes the Heinz body. Heinz bodies are commonly seen in Hemoglobin H disease (Thalassemia intermedia), G6PD deficiency, and other thalassemias. Choice E (reciprocal translocation between chromosome 9 and chromosome 22) describes the Philadelphia chromosome with isclassically associated with chronic myelogenous leukemia.

A 43-year old male undergoes cholesterol screening and is found to have an abnormal cholesterol panel. After a trial of 6 months of rigid diet and exercise, his repeat cholesterol values are as follows: HDL 49 mg/dL(> 45 mg/dL) LDL 190 mg/dL (< 100 mg/dL) TRIGLYCERIDES 149 mg/dL (< 150 mg/dL) Which of the following medications is the most appropriate management for this patient? a. simvastatin b. cholestyramine c. ezetimibe d. fenofibrate e. nicotinic acid

Choice A (simvastatin) is the correct answer. The patient has an elevated low density lipoprotein levels. HMG - CoA reductase inhibitors ("statins") are the best drugs to lower LDL levels and have been shown to reduce cardiovascular morbidity and mortality. (cholestyramine) is a bile acid sequestrant that can also be used to lower LDL but it is not the most effective. They also have the side effect of increasing triglyceride levels so it may not be the ideal medication to use in this patient with borderline triglyceride levels. Choice C (ezetimibe) is also used to lower LDL but it is not the most effective LDL-lowering drug. Choice D (fenofibrate) is the best drug to lower triglyceride levels. It has minimal LDL lowering properties. Choice E (nicotinic acid) is the best drug to increase HDL (high density lipoprotein) levels. It

Which of the following is most specific for differentiating schizophrenia from mania? a. inflated self esteem b. flat, blunted, emotional affect C. disorganized speech and thinking d. expansive or irritable mood e. impairment of social function

Choice B (flat, blunted emotional affect) is a negative symptom of schizophrenia that would not be seen in mania. All of the other choices can possibly be seen in both disorders

A 40-year-old male is complaining of exquisite pain and swelling to the right foot for 1 day. The patient states the pain started the day after Thanksgiving. He admits to having consumed large amounts of steak and beer for the holidays. On physical examination, the first metatarsophalangeal joint is tender, red, warm and edematous. A right foot radiograph is taken: Which of the following would be the most likely arthroscopic finding in this patient? a. less than 200 white blood cell/mm b. 21,000 WBC (90% PMN), negatively birefringent, needle shaped crystals c. 21,000 WBC (90% PMN), positively birefringent rhomboid shaped crystals d. 59,000 WBC (90% PMN), cloudy fluid e. 35,000 WBC (90% PMN) no crystals seen

Choice B (21,000 WBC (90% PMN), negatively birefringent, needle shaped crystals) is the correct answer. The presentation is classic for gouty arthritis. Gouty arthritis is due to uric acid deposition in the joints. Exacerbations are frequently associated with the consumption of meat, seafood and alcohol (especially beer). Podagra (involvement of the 1% metatarsophalyngeal joint) is classic for gout. Over time, inflammatory changes may cause bony destruction, leading to the classic radiologic "rat bite" erosions seen on this radiograph (at the first metatarsal head) Choice A (less than 200 white blood cell/mm) is considered a normal joint fluid value. Choice C (21,000 WBC (90% PMN), positively birefringent rhomboid shaped crystals) is classic for pseudogout. Choice D [59,000 WBC (90% PMN), cloudy fluid) is seen with septic arthritis. A white blood cell count in the joint fluid >50,000 is highly suspicious of septic arthritis. Choice E [35,000 WBC (90% PMN) no crystals) is consistent with any condition that causes inflammatory arthritis.

A male is being evaluated. He states that he was a relatively normal height before puberty. After puberty, he developed a tall, thin stature. He has had issues in school with language comprehension. He later became obese. On physical examination, he has long limbs, gynecomastia and testicular atrophy as well as the presence of ataxia and scoliosis. Which of the following is the most likely diagnosis? a. Marfan syndrome b. Klinefelter's syndrome c. Turner's syndrome d. Ehlers-Danlos Syndrome e. Hypopituitarism

Choice B (Klinefelter's syndrome) is the correct answer. Klinefelter's syndrome (47, XXY) is due to an extra X chromosome. They are considered genetically male but may have a male, female or intersex phenotype. These patients often have gynecomastia & testicular atrophy. They also tend to have a tall thin stature but become obese later in life and issues with language comprehension. Choice A (Marfan syndrome) is an autosomal dominant, genetic connective tissue disorder that leads to cardiovascular, ocular & musculoskeletal findings in addition to multi-systemic involvement. Choice C (Turner's syndrome) 45,X is due to the absence of all or part of a sex chromosome. This leads to gonadal dysgenesis & primary amenorrhea. They often develop premature ovarian failure. Classic findings include: short stature, webbed neck, prominent ears and a broad chest with hypoplastic, widely-spaced nipples. Choice D (Ehlers-Danlos Syndrome) is a genetic disorder of collagen synthesis leading to skin hyperextensibility, fragile connective tissue & joint hypermobility. Choice E (Hypopituitarism) is associated with decreased production of follicle stimulating hormone, luteinizing hormone, adrenocorticotrophic hormone, thyroid stimulating hormone and somatotropin.

Which of the following is the most likely diagnosis based on the rhythm strip? 1S thhhhhhhh R a. First degree heart block b. Second degree heart block (Mobitz I) c. Second degree heart block (Mobitz II) d. Third degree heart block e. Ashman phenomenon

Choice B (Second degree heart block (Mobitz 1)] is the correct answer. Mobitz 1 second degree heart block (Wenckebach) is diagnosed by progressive lengthening of the PR interval with occasional nonconducted ("dropped") QRS complexes. In this rhythm strip, the second and seventh atrial impulses (P waves) were not conducted to the ventricles (absent QRS complexes). Choice A (First degree heart block) is associated with a prolonged PR interval (greater than 0.20 seconds) but every P wave is followed by a QRS complex because all of the atrial impulses are conducted to the ventricles. Choice C [Second degree heart block (Mobitz 11)] is associated with a PR interval of constant duration and occasional nonconducted impulses ("dropped' QRS complexes). The distinguishing factor between Type I and Type II is that in type I, the PR interval progressively lengthens before the nonconducted QRS whereas the PR interval remains the same duration in type Il before the nonconducted QRS. Choice D (Third degree heart block) is associated with atrioventricular dissociation. The P waves occur at regular intervals, the QRS complexes also occur at a regular intervals but the two are not connected (they operate independently of each other). Choice E (Ashman phenomenon) is seen in patients with atrial fibrillation, which are occasional wide complex beats that occur with a right bundle branch morphology (due to aberrant conduction).

Which of the following accurately describes the primary pathophysiology of rheumatoid arthritis? a. degenerative wear and tear of the joint, leading to asymmetric joint narrowing b. T cell-mediated pannus formation with symmetric joint destruction and symmetric narrowing c. Joint damage due to enthesitis d. Imbalance in cartilage repair leading to more cartilage destruction than repair e. Joint damage as a result of peripheral neuropathy and repetitive microtrauma

Choice B (T cell-mediated pannus formation with symmetric joint destruction and narrowing) is the correct choice. This is the classic description of the joint destruction associated with rheumatoid arthritis. Choice A (degenerative wear and tear changes leading to asymmetric joint narrowing) and Choice D (Imbalance in cartilage repair leading to more cartilage destruction than repair) describes the pathophysiologic changes associated with osteoarthritis. Choice C (Joint damage due to enthesitis) describes the pathophysiology of seronegative spondyloarthropathies such as reactive arthritis, psoriatic arthritis, and ankylosing spondylitis. Choice E (Joint damage as a result of peripheral neuropathy and repetitive microtrauma) classically describes the pathophysiology of Charcot's joint associated with diabetes mellitus.

In the evaluation of a 32-year-old female at 19 weeks gestation, a triple screen test is done and shows the following: Alpha-fetoprotein: low Beta hCG: high Estradiol: low Which of the following is the most likely cause of these findings? a. Open neural tube defects b. Trisomy 18 C. Trisomy 21 d. Multiple gestation e. Underestimation of gestational age

Choice C (Trisomy 21) is the correct answer. In the triple screen test a low alpha-fetoprotein, a high beta HCG and a low estradiol is associated with a high likelihood of trisomy 21. Choice A (open neural tube defects), Choice D (multiple gestation) and choice E (underestimation of gestational age) are all associated with a high alpha fetoprotein. Choice B (Trisomy 18) is associated with low alpha fetoprotein, low estradiol and low beta HCG levels. Infants with Trisomy 18 are often stillborn or die within the first year of life.

A 43-year-old male went camping where he hunted and ate rabbits. 5 days later, he develops a headache, fever and nausea. On his hand, there is an ulcerated papule with a central eschar and tender regional lymphadenopathy. There is also splenomegaly and diarrhea. Which of the following is the most likely diagnosis? a. Cutaneous anthrax b. Tularemia c. Coccidiodomycosis d. Leishmaniasis e. Brucellosis

Choice B (Tularemia) is the correct answer. Tularemia is caused by Francisella tularensis, a gram-negative coccobacilli. It is transmitted via a tick or insect bite or from handling rodent and rabbit tissues Choice A (Cutaneous anthrax) is associated with a black eschar. It is normally found in cattle, horses, goats, sheep and swine. It presents with an erythematous papule that ulcerates into a black eschar with surrounding edema and vesicles. Choice C (Coccidiodomycosis) may also present with an eschar. It grows in the soil of arid/desert regions in Southwestern United States, Mexico, South and Central America. Choice D (Leishmaniasis) may also present with an eschar. It is most commonly associated with bites from a female sand fly. It presents with small erythematous papules. Choice E (Brucellosis) is not classically associated with eschar formation.

A 53-year-old male has had chronic, intermittent, non-bloody diarrhea for the last 2 months associated with intermittent fever. He has had intermittent joint pains and describes a weird occurrence of twitching of his eye when he is chewing. Transglutaminase antibodies are negative. A small bowel duodenal biopsy is periodic acid-Schiff (PAS) stain positive and is positive for the presence of dilated lacteals. Which of the following is the most likely diagnosis? a. seronegative spondyloarthopathies b. Whipple's disease c. Crohn's disease d. ulcerative colitis e. Giardiasis

Choice B (Whipple's disease) is the correct choice. Diarrhea, joint pain, fever are commonly seen in patients with Whipple's disease. CNS involvement may cause twitching of the eye muscles while chewing. Positive PAS macrophages and dilation of the lacteals on biopsy are hallmark of the disease. Choice A (seronegative spondyloarthropathies) can cause joint pain but diarrhea is not a common occurrence in these patients. Choice C (Crohn's) disease can be associated with the symptoms. Biopsy would show transluminal inflammation. Choice D (Ulcerative colitis) classically is associated with bloody diarrhea and is associated with diffuse erythema of the mucosa and submucosa. Choice E (Giardiasis) can cause a chronic diarrhea but is not associated with Positive PAS macrophages.

A 32-year-old woman at 35 weeks gestation presents to the emergency room with painful contractions. A pelvic examination is done, revealing 4 cm cervical dilation with 81% effacement. There is a positive fern test and the L:S ratio is <2:1. There are no signs of fetal distress upon testing. She is afebrile and there is no uterine tenderness or vaginal discharge. Which of the following is the most appropriate management at this time? a. administration of prostaglandins b. administration of magnesium sulfate and corticosteroids c. administration of terbutaline d. observation until 85% effacement e. administration of oxytocin to induce labor

Choice B (administration of magnesium sulfate and corticosteroids) is the correct answer. The positive fern test and cervical effacement suggests preterm labor (defined as regular uterine contractions and cervical dilation before 37 weeks gestation). An L:S ratio <2:1 indicates fetal lung immaturity so tocolytics (such as terbutaline or magnesium sulfate) can be given for 48 hours to delay delivery and allow for corticosteroid administration (to enhance fetal lung maturity). This is not used if there is suspected infection (in this vignette there is no foul discharge and the patient is afebrile) Choice A (administration of prostaglandins) and choice E (oxytocin) would enhance uterine contraction and increase progression of premature labor. Choice C (terbutaline) is a tocolytic, which can be used to delay premature labor, but the L:S ratio presented in this vignette makes the addition of the steroid and the tocolytic (Choice B) a more effective choice. Choice D (observation until 85% effacement) is not the correct choice as the patient is already in premature labor.

Pulmonary fibrosis and thyroid disorders are classic side effects of which of the following medications? a. propranolol b. amiodarone c. adenosine d. procainamide e. verapamil

Choice B (amiodarone) is the correct answer. Amiodarone is a class III antiarrhythmic. Because it contains iodine, it may cause hyperthyroid or hypothyroid disorders. Other side effects include corneal deposits with long term use and pulmonary fibrosis. Choice A (propranolol) is a nonselective beta 1 and beta 2 antagonist. Side effects of beta blockers include depression, fatigue, impotence, second or third degree heart block, bradycardia, masking of the symptoms of hypoglycemia, bronchoconstriction in patients with asthma and chronic obstructive pulmonary disorders as well as unopposed alpha constriction if given to patients with cocaine-induced myocardial infarctions. Choice C (adenosine) is the drug of choice in patients with supraventricular tachycardia and narrow complex tachycardia. Side effects include transient flushing, chest pressure/pain and bronchospasm. Choice D (procainamide) is a class | antiarrhythmic that has been shown to prolong repolarization and the refractory period. Class | antiarrhythmics block sodium channels. Side effects include torsades de pointes, hypotension, tachycardia, tinnitus, and kidney injury. Choice E (verapamil) is a nondihydropyridine calcium channel blocker. Side effects of calcium channel blockers include constipation, dizziness, bradycardia, second or third degree heart block and peripheral edema.

A 30-year-old woman with a history of Raynaud's Phenomenon presents to the clinic with difficulty swallowing. An esophagram shows esophageal motility abnormalities. On physical exam, she has clawed hands and some calcified lesions on her shins. Her skin is thin and shiny on the face and neck as well as distal to the elbows and the knees. Which of the following antibodies would most likely be seen in this patient? a. anti-La antibodies b. anti-centromere antibodies C. anti-smooth muscle antibodies d. anti-Mi2 antibodies e. anti-SRP antibodies

Choice B (anti-centromere antibodies) is the correct answer. This is the classic description of CREST syndrome (Limited scleroderma). CREST is a mnemonic for Calcinosis cutis, Raynaud's phenomenon, Esophageal motility disorders, Sclerodactyly & Telangiectasia. Anti-centromere antibodies are associated with limited scleroderma whereas Scl-70 antibodies are more commonly associated with diffuse scleroderma.

Which of the following is increased in patients with emphysema-dominant chronic obstructive pulmonary disease? a. Diffusing capacity of the lung for carbon monoxide (DLCO) b. Forced vital capacity (FVC) c. Forced expiratory volume in 1 second (FEV1) d. Residual volume (RV) e. Alpha-1 antitrypsin levels

Choice D (residual volume) is correct. Obstructive diseases (such as COPD, Bronchiectasis) are associated with increased lung volumes (such as residual volume), reflecting air trapping (obstruction). All the other choices are decreased in emphysema.

Depression is a side effect of which of the following anti hypertensive medications? a. ACE inhibitors b. Beta blockers c. Hydrochlorothiazide d. Angiotensin II receptor antagonist e. Hydralazine

Choice B (beta blockers) is the correct answer. Side effects of beta blockers include fatigue, depression, impotence, hypotension, bradycardia, second or third degree heart block, and bronchospasms in patients with reactive airway diseases such as asthma & COPD. Beta blockers may also worsen some peripheral vascular disease states, such as Raynaud's phenomenon. They may mask the symptoms of hypoglycemia (tachycardia, tremor and sweating). A (ACE inhibitors) side effects include: teratogenicity, hypotension (it is used as an anti-hypertensive). Although ACE inhibitors are "renoprotective", they may cause azotemia in some patients with baseline kidney disease. Dry cough and angioedema are common side effects due to an increase in bradykinin, a potent vasodilator (as a result of ACE inhibition). The inhibition of aldosterone may lead to hyperkalemia. Choice C (Hydrochlorothiazide) side effects include hyponatremia, hypokalemia, mild cholesterol elevations, hyperuricemia, hyperglycemia, hypercalcemia, sulfa allergies and metabolic alkalosis. Choice D (Angiotensin II receptor antagonist) side effects include: hyperkalemia and teratogenicity. Choice E (Hydralazine) side effects include: tachycardia, gastrointestinal symptoms, drug-induced lupus, neutropenia, myocardial infarction, blood dyscrasias and peripheral neuritis.

A 43-year-old male was ejected off of his motorcycle and had a witnessed brief loss of consciousness. He presents to the emergency room with headache, nausea, vomiting and dizziness. A CT scan of the head shows no evidence of fracture, midline shift, or evidence of intracranial bleed. He is not on aspirin or other anticoagulants. He remains lucid during his emergency room visit and upon discharge. Which of the following is the recommended management of this patient? a. raising the head of the bed, placing the patient in a dark, quiet room, supplemental oxygen and phenytoin seizure prophylaxis. b. cognitive and physical rest c. administer dexamethasone and mannitol d. immediate neurosurgery consult e. administer hypertonic fluids to reduce the neurologic symptoms

Choice B (cognitive and physical rest) is the correct answer. The patient has trauma-induced neurological symptoms of altered mental status changes and loss of consciousness consistent with concussion syndrome since the head CT is negative. Cognitive and physical rest is the main management of concussion syndrome Choice A (raising the head of the bed and placing the patient in a dark, quiet room), Choice C (administer dexamethasone and mannitol), Choice D (immediate neurosurgery consult) are the management of subarachnoid hemorrhage. This patient remains lucid and has a negative CT scan. If there were a high suspicion for subarachnoid hemorrhage, then a lumbar puncture would be indicated to look for blood in the CSF. Choice E (administer hypertonic fluids to reduce the neurologic symptoms) in addition to furosemide is the management of choice for severe, symptomatic hyponatremia.

Which of the following is the first line management of warm antibody type autoimmune hemolytic anemia in a patient with systemic lupus erythematosus? a. Avoidance of the cold b. Corticosteroids C. Splenectomy d. Deferoxamine e. Hydroxyurea

Choice B (corticosteroids) is the correct answer. Corticosteroids are the first line agents in warm antibody autoimmune hemolytic anemia. Other immunosuppressant medications may also be used as well. Splenectomy (Choice C) may be used in refractory cases (as it removes the site of red blood cell destruction). Choice A (avoidance of the cold) is the treatment of cold antibody autoimmune hemolytic anemia. Choice D (Deferoxamine) is chelation therapy used in states of iron overload. Choice E (Hydroxyurea) can be used in severe pain in sickle cell crisis and in long-term management in some cases to reduce the frequency of pain crisis in patients with sickle cell anemia

A 17-year-old newly diagnosed type I diabetic is placed on regular insulin and NPH insulin. His finger sticks in the morning before breakfast have been consistently elevated. He is instructed to record his fingersticks for the next 3 days and the fingerstick glucose levels are as follows: 11 PM (89,90,92) 2AM (40.52,57) 6AM (180,240,278) 9AM after breakfast (98,101,101) (Normal fasting levels 80-110 mg/dL. Normal Post prandial levels <140 mg/dL) Which of following is the most appropriate management of this patient? a. moving his NPH dose to the afternoon b. decreasing his bedtime NPH dose C. decreasing his evening regular insulin dose d. moving his last regular insulin dose from evening to bedtime e. avoiding a bedtime snack

Choice B (decreasing his bedtime NPH dose) is the correct answer. This vignette describes the Somogyi effect. The Somogyi effect can be seen in patients who are started on insulin. It is characterized by nocturnal hypoglycemia (note the 2am low serum glucose levels) followed by a rebound hyperglycemic overshoot. The Somogyi effect is reduced by preventing the hypoglycemia by either decreasing the bedtime NPH dose, moving the NPH dose earlier or giving the patient a bedtime snack Choice A (moving his NPH dose to the afternoon) wouldn't affect the overnight hypoglycemia as the effects NPH peaks in 12 hours. Choice C (decreasing his evening regular insulin dose) would be correct if the postprandial glucose levels were elevated. Choice D (moving his last regular insulin does from evening to bedtime) wouldn't have a great effect on the 2 AM levels, as regular insulin peaks at about 2-3 hours. Choice E (avoiding a bedtime snack) is used in the management of patients with the Dawn phenomenon. The Dawn phenomenon is caused by decreased insulin sensitivity and the nightly surge of counter regulatory hormones during overnight fasting.

A 54-year-old female is seen in the clinic for refractory hypertension. On physical examination, she has truncal obesity with thin arms and legs, striae on her abdomen and supraclavicular fat pads with facial redness and roundness. Which of the following is the most common etiology of her symptoms overall? a. Cushing's disease b. Exogenous corticosteroid use C. Adrenal tumor d. Ectopic ACTH-producing tumor e. Ketoconazole use

Choice B (exogenous corticosteroid use) is the correct answer. The vignette is describing the classic findings associated with Cushing syndrome (hypercortisolism). The most common cause overall is exogenous steroid use. Choice A (Cushing's disease) is Cushing's syndrome due to increased pituitary ACTH secretion. Although this is the most common cause of ENDOGENOUS Cushing's syndrome, corticosteroid use is the most common cause overall. Cushing's disease causes 70% of endogenous causes. Endogenous causes include Cushing's disease, adrenal tumor (Choice C) and ectopic ACTH-producing tumor (Choice D). Many people confuse Cushing's syndrome and Cushing's disease. Cushing's disease one of the causes of Cushing's syndrome. Choice E ( Ketoconazole) is an antifungal medication that decreases cortisol production, so it used to medically treat Cushing's syndrome, it doesn't cause it.

A 43-year-old male presents with episodes of feeling as if the "room is spinning" that lasts for minutes to several hours over the last 2 days. This sensation is often accompanied with hearing loss, a ringing sensation in the ear and describes voices sounding as if he is "underneath water". Which of the following physical examination findings would most likely be present in this patient? a. bullae on the tympanic membrane b. horizontal nystagmus c. vertical nystagmus d. non fatigable nystagmus e. granulation tissue seen on the tympanic membrane

Choice B (horizontal nystagmus) is the correct choice. The 4 cardinal symptoms of Meniere's disease are episodic peripheral vertigo, ear fullness, hearing loss and tinnitus. Peripheral vertigo causes horizontal nystagmus. In peripheral vertigo, the nystagmus is usually fatigable. . Choice A (bullae on tympanic membrane) is classically seen with Mycoplasma pneumoniae infections. Choice C (vertical nystagmus) is seen with central vertigo. Central vertigo is also associated with non fatigable nystagmus (Choice D). Causes include: brainstem or cerebellar lesions, migraines and multiple sclerosis. Choice E (granulation tissue seen on the tympanic membrane) is associated with cholesteatomas. Cholesteatomas are associated with conductive hearing loss, peripheral vertigo and otorrhea.

A 37-year-old male with a history of chronic hepatitis B and Wilson's disease (for which he takes pencillamine) presents to the emergency room with shortness of breath. He denies any recent skin or throat infections. Physical examination reveals dullness to percussion, decreased fremitus and decreased breath sounds at the bases bilaterally with no crackles heard throughout the lung fields. The heart has a regular rate and rhythm. S1 and 52 are normal. No S3, S4, murmurs, rubs or other sounds are heard. There is bilateral peripheral and sacral edema. The labs are consistent with a hypotonic hypervolemic hyponatremia and hyperlipidemia. A urinalysis reveals proteinuria, oval fat bodies and fatty casts. A 24-hour urine protein collection shows proteinuria of 4 grams. Which of the following would most likely be seen on biopsy? a. IgA deposits in the glomerulus b. normal cellularity with thickened glomerular basement membrane c. nodular glomerulosclerosis with pink hyaline material around the glomerular capillaries d. hypercellularity with increased monocytes and positive immune humps e. hypercellularity with the presence of crescent-shaped collapse of the Bowman's capsules

Choice B (normal cellularity with thickened glomerular basement membrane) is the correct answer. Membranous nephropathy, which is one of the most common causes of primary nephrotic syndrome in adults, can often be caused by viral hepatitis (this patient has hepatitis B). Pencillamine can also cause membranous nephropathy. Membranous nephropathy is classically associated with basement membrane thickening on kidney biopsy. Nephrotic syndrome is characterized by proteinuria, hypoalbuminemia, edema and hyperlipidemia. The decreased oncotic pressure (from the hypoalbuminemia) may cause a transudative pleural effusion. Choice A (IgA deposition in the glomerulus) is classically associated with Berger's disease associated glomerulonephritis. It would be associated with dysmorphic red blood cells, red blood cell casts, and hypertension. Choice C (nodular glomerulosclerosis with pink hyaline material around the glomerular capillaries) is the classic description of the Kimmelstiel-Wilson lesion, which is pathognomonic of diabetes mellitus-induced nephropathy. Choice D (hypercellularity with increased monocytes and positive immune humps) is classically seen with post streptococcus glomerulonephritis. Glomerulonephritis is associated with hypercellularity (increased inflammatory cells) hematuria, the presence of red blood cell casts and dysmorphic red blood cells. Choice E (hypercellularity with the presence of crescent-shaped collapse of the Bowman's capsules) is classically associated with rapidly progressing glomerulonephritis. Glomerulonephritis is associated with hematuria and red blood cell casts.

A 66-year-old male with chronic bronchitis has symptoms consistent with cor pulmonale. Which of the following medications have been shown to reduce overall mortality? a. Theophylline b. Oxygen c. Ipratropium d. Flunisolide e. Albuterol

Choice B (oxygen) is the correct answer. In chronic bronchitis, chronic hypoxemia leads to hypoxemia-mediated vasoconstriction of the pulmonary bed, leading to pulmonary hypertension and increased afterload on the right side. The right side of the heart has to work against this pulmonary hypertension, causing right ventricular hypertrophy and eventually right sided heart failure. Oxygen has been shown to decrease mortality by reducing the hypoxemia-mediated vasoconstriction, decreasing the workload of the right side of the heart. All the other medications can be used but are not associated with a reduction of mortality in patients with cor pulmonale.

A 4-year-old boy is brought to the pediatric clinic by his mother after he developed a high fever, cough, conjunctivitis and a runny nose. He later developed small red spots in the buccal mucosa with a pale blue/white center opposite his first and second molars. A brick-red rash began at his hairline and face, progressing to his extremities. The rash lasts 7 days. Which of the following is the most likely diagnosis? a. rubella b. rubeola C. fifth's disease d. mumps e. hand foot and mouth disease

Choice B (rubeola) is the correct answer. Rubeola is the causative agent of measles. The vignette describes Koplik's spots. The rash of measles is classically described as a brick-red rash starting on the face and spreading to the trunk, lasting 7 days. It is often preceded by a prodrome of the 3 Cs: cough, coryza and conjunctivitis. Choice A (rubella) is associated with a light red to pink spotted rash that lasts about 3 days. Choice C (fifth's disease) otherwise known as erythema infectiosum is associated with coryza, fever and the appearance of an erythematous rash with circumoral pallor giving the classic "slapped cheek" appearance. This rash is often followed by a lacy, reticular rash on the extremities (especially the upper extremities and the rash often spares the palms and the soles). Choice D (mumps) is associated with parotid gland swelling. Choice E (hand foot and mouth disease) classically presents with vesicular lesions on a reddened based with an erythematous halo in the oral cavity followed by vesicular rash involving the palms and the soles.

A 43 year old previously healthy female presents to the emergency room with chest pain and palpitations the symptoms continue despite oxygen and IV fluids therapy her blood pressure is 80 / 60 she is diaphoretic dizzy and unable to speak in full sentences her pulses are palpable but rapid an EKG is performed showing a regular narrow complex tachycardia at 180 beats per minute. There are no St or T wave changes consistent with myocardial infarction and the patient has no significant cardiac risk factors which of the following is a recommended management of this patient? A atropine b. synchronized cardioversion C adenosine D amiodarone E. unsynchronized cardioversion

Choice B (synchronized cardioversion) is the correct answer. The patient has an unstable tachycardia, evident by chest pain, hypotension and persistent symptoms. Synchronized cardioversion is the first line management of unstable tachycardia. Synchronized cardioversion works by giving an electrical shock, which places all the cardiac cells in the absolute refractory period, terminating the tachyarrhythmia and allowing recovery of the sinoatrial node to become the dominant pacemaker. Choice A (Atropine) is used as the first line management for symptomatic bradycardia. anticholinergic drug that will increase the heart rate. Choice C (Adenosine) can be used as first line management for stable, regular, narrow complex tachycardia, such as paroxysmal supraventricular tachycardia. Since the patient is unstable, this would not be the ideal choice. Choice D (Amiodarone) is a class III antiarrhythmic (with class I through IV properties). It is used as first line management for stable, wide complex tachycardia. Choice E (Unsynchronized cardioversion) otherwise known as defibrillation is indicated in only 2 instances: ventricular tachycardia without a pulse or ventricular fibrillation.

A 42-year-old male with a history of schizophrenia is being managed with Haloperidol (Haldol). He develops lip smacking, teeth grinding and rolling of the tongue. There are no signs of tremor, slowness of movement or other symptoms. Which of the following is the most likely diagnosis? a. neuroleptic malignant syndrome b. tardive dyskinesia c. acute dystonic reaction d. serotonin syndrome e. akathisia

Choice B (tardive dyskinesia) is the correct answer. The symptoms in this vignette are consistent with tardive dyskinesia, a complication of long-term dopamine blocking agents such as Haloperidol. Symptoms include repetitive involuntary movements, mostly involving the extremities and the face. Symptoms include lip smacking, teeth grinding and rolling of the tongue. Choice A (Neuroleptic malignant syndrome) is a condition seen due to decreased dopamine activity in patients on dopamine antagonists (such as anti-psychotic medications). It can also present with hyperthermia but these patients tend to be hyporeflexive, and often develop urinary incontinence. Choice C (acute dystonic reaction) describes the reversible extrapyramidal symptoms that usually occur in hours to days after the initiation of a dopamine-blocking anti-psychotics (especially 1st generation anti-psychotics). The extrapyramidal symptoms include: intermittent spasms, trismus, protrusions of the tongue, facial grimacing, torticollis, and difficulty speaking. Choice D (serotonin syndrome) is a potentially life-threatening condition due to increased serotonin activity. It is usually due to the potentiation between 2 serotonin agonistic drugs (including the natural herbal antidepressant St. John's Wort). Serotonin syndrome is characterized by autonomic dysfunction including tachycardia, blood pressure fluctuations, hyperthermia, hyperreflexia, myoclonus, dilated pupils, agitation and a hyperactive GI tract (due to the effect of serotonin on the enterochromaffin-like cells). Management includes serotonin antagonists such as cyproheptadine and benzodiazepines to decrease muscle contraction-induced hyperthermia. Choice E (akathisia) describes the extrapyramidal symptoms of restlessness and the need to be in constant motion. Dopamine-blocking agents can lead to the development of akathisia.

A 38-year-old male with a history of Type Il diabetes mellitus presents to the clinic with sudden onset of slurred speech, drooping of the corner of his right mouth & inability to close the right eye fully. He also states that he had ear pain for a day that preceded the symptoms. The symptoms are not associated with visual changes, nystagmus or drop attacks. On physical examination, he has full range of motion with 5/5 motor & sensation of both the upper and lower extremities. Which of the following is the most likely diagnosis? a. right anterior cerebral artery occlusion b. right posterior cerebral artery occlusion c. Bell's palsy d. left anterior cerebral artery occlusion e. transient ischemic attack

Choice C (Bell's palsy) is the correct answer. Bell's palsy is an idiopathic, unilateral facial paralysis due to cranial nerve VII (Facial nerve) involvement. It classically presents with ipsilateral hyperacusis (which may be absent) followed by the onset of unilateral facial paralysis and loss of the anterior 2/3 of taste (because the facial nerve has both motor and sensory functions). Since only the facial nerve is involved, the trunk and the extremities should not be affected Choice A (right anterior cerebral artery occlusion) and choice D (left anterior cerebral artery occlusion) would result in contralateral hemiparesis (weakness) that tends to be greater in the leg than the arm. It is also associated with urinary incontinence, personality changes and abulia (lack of will). Choice B (right posterior cerebral artery occlusion) can result in visual hallucinations, and "crossed symptoms" - meaning ipsilateral cranial nerve deficits, contralateral muscle weakness, comas and drop attacks. This patient only has facial involvement, making posterior cerebral artery occlusion less likely. Choice E (Transient ischemic attack) is an acute neurologic deficit involving the brain, spinal cord or the optic nerve and usually leads to clinical manifestations corresponding to one of the carotid arteries (such as amaurosis fugax) or the cerebral arteries. Symptoms classically resolve within 24 hours.

A 23-year-old male presents to the emergency room with high fever, headache, joint pain and chest pain. His ECG is positive for second-degree heart block, which was not present in prior ECGs. A lumbar puncture is performed and is consistent with Lyme meningitis. Which of the following is the treatment of choice? a. Oral Amoxicillin b. IV Ampicillin/sulbactam C. IV Ceftriaxone d. IV Gentamicin e. Oral Doxycycline

Choice C (Ceftriaxone) is the correct choice. IV Ceftriaxone is used in Lyme disease in patients who have severe disease, neurologic disease (other than facial palsy), and cardiac disease (such as second or third degree heart blocks). Other drugs used in the management of neurologic Lyme include cefotaxime and penicillin G. Choice A (Amoxicillin) can be used in all other cases of Lyme disease (except noted above). Amoxicillin is the drug of choice in children under the age of 8 years (the tetracyclines can cause permanent staining of the teeth in children under the age of 8). Choice E (Oral Doxycycline) is the drug of choice for Lyme disease in patients older than 8 years old in all other cases of Lyme disease than noted above in Choice C.

A 32-year-old sexually active male develops the following painless lesion: Which of the following is used in the diagnostic evaluation of this patient? a. India ink b. Tzanck smear C. Darkfield microscopy d. Potassium Hydroxide e. Polymerase chain reaction

Choice C (Darkfield microscopy) is the correct answer. The picture in this vignette depicts a chancre, the primary lesion of syphilis infection. Treponema pallidum is a spirochete and the causative agent of syphilis. It is very difficult to grow Treponema pallidum in culture. Darkfield microscopy is a direct method to diagnose syphilis in the presence of a chancre or condyloma lata. An indirect method of using the rapid plasma reagent screening An indirect method of using the rapid plasma reagent screening test (RPR) with a confirmatory fluorescent treponemal antibody absorption test (FTA-ABS) are used in all other presentations of syphilis. Choice A (India ink) is used in the diagnosis of Cryptococcus neoformans. Other methods used in the diagnosis of Cryptococcal infections include Cryptococcal antigen (and blood cultures in cases of disseminated infection). Choice B (Tzanck smear) can be used in the diagnosis of Varicella Zoster and Herpes simplex virus infections. Choice D (Potassium Hydroxide) can be used in the diagnosis of candida fungal infections, or as part of the diagnostic vaginal mount testing in patients suspected of having bacterial vaginosis, trichomoniasis or vaginal candidiasis. Choice E (Polymerase chain reaction) is used in many disorders, such as varicella zoster infections.

. Which of the following describes myoclonus? a. rapid, involuntary, jerky, uncontrolled purposeless movements b. repetitive, non-rhythmic movements or vocals c. brief, sporadic single repetitive jerks or twitching of 1 or more muscle groups d. sustained contraction with twisting of the body and abnormal posturing e. repetitive, rhythmic jerking that usually lasts less than 2 to 3 minutes

Choice C (brief, sporadic single repetitive jerks or twitching of 1 or more muscle groups) is the correct answer. Myoclonus is an extrapyramidal symptom but can also be seen when a patient is falling asleep. Hiccups is a form affecting the diaphragm Choice A (rapid, involuntary, jerky, uncontrolled purposeless movements) describes chorea, which is another type of extrapyramidal symptom. Choice B (repetitive, non-rhythmic movements or vocals) describes tics. Choice D (sustained contraction with twisting of the body and abnormal posturing) describes dystonia, which is due to sustained contraction of antagonist muscles. Writer's cramp is an example of dystonia. Choice E (repetitive, rhythmic jerking that usually lasts less than 2 to 3 minutes) describes clonus. Clonus is rhythmic, myoclonus is not rhythmic.

A 19-year-old female presents to the emergency room with a sudden onset of blanchable, edematous, pink papules that forms irregular wheals. The patient states that on the way to the emergency room, some of the lesions disappeared and new lesions appeared. Which of the following signs are classically associated with this condition? a. Auspitz sign b. Koebner's phenomenon C. Dermatographism d. Nikolsky's sign e. Koplik's spots

Choice C (Dermatographism) is the correct answer. Urticaria is a type 1 hypersensitivity reaction associated with IgE and increased mast cells in the skin. It classically presents with blanchable edematous pink papules that form irregular wheals. These wheals may disappear, with the appearance of new lesions Choice A (Auspitz sign) is classically associated with plaque psoriasis. Auspitz sign is punctate bleeding (spots) when the plaques of psoriasis are unroofed. Choice B (Koebner's phenomenon) is new skin lesions appearing along the lines of trauma. Koebner's phenomenon can be seen in many diseases: psoriasis, lichen planus, vitiligo, and some infectious diseases. Choice D (Nikolsky's sign) is sloughing off of the epidermis when slight pressure is applied to the skin. A positive Nikolsky's sign can be seen in toxic epidermal necrolysis, Steven-Johnson syndrome, pemphigus vulgaris, and staphylococcal scalded skin syndrome. Choice E (Koplik's spots) are clustered, white lesions on the buccal mucosa with an erythematous rim most commonly seen opposite the lower 1% and 2nd molars. They can be seen with rubeola (measles).

A 31-year-old female had a normal vaginal delivery at 32 weeks gestation. 24 hours later, she has 3 episodes of uterine bleeding. She is afebrile with stable vital signs. There is no foul-smelling discharge from the vagina or uterine tenderness. She does not have any bruising of the skin. Prothrombin time and partial thromboplastin time are within normal limits. Which of the following is the most appropriate treatment at this time? a. fresh frozen plasma b. IV Ceftriaxone C. IV oxytocin d. IV amoxicillin + gentamicin + metronidazole e. Obtain D-dimer levels

Choice C (IV oxytocin) is the correct answer. The most common cause of post partum hemorrhage (up to 80%) is uterine atony (loss of uterine muscle tone). After delivery, uterine muscle contraction causes blood vessel compression and reduction of uterine blood flow. Oxytocin causes rhythmic uterine contraction, improving uterine tone and reduces bleeding Choice A (fresh frozen plasma) is used in patients severe coagulation protein deficits when the specific factor needed is unavailable. It may also be used in conjunction with plasmapheresis. Choice B (IV ceftriaxone) is not used for post partum hemorrhage without signs of infection. Choice D (IV amoxicillin + gentamicin + metronidazole) is used in the management of chorioamnionitis. The absence of uterine tenderness, fever or foul smelling discharge makes this diagnosis unlikely. Choice E (obtain D-dimer levels) is indicated in patients with low suspicion of pulmonary embolism.

A 45-year-old male is being evaluated for a chronic cough. A chest radiograph is obtained, which shows honeycombing consistent with idiopathic pulmonary fibrosis. Which of the following pulmonary function test findings is most consistent with restrictive pulmonary disease? a. Increased total lung capacity b. Increased residual volume c. Increased FEV1/FVC ratio d. Increased lung compliance e. Increased functional residual capacity

Choice C (Increased FEV1/FVC ratio) is the correct answer. Idiopathic pulmonary fibrosis is a type of restrictive disorder. The scarring (fibrosis) "restricts" the movement of the lung. In restrictive disorders, both the forced expiratory volume in one second (FEV1) and forced vital capacity (FVC) are decreased. But the decrease in FVC is more than that of FEV1, resulting in a normal or higher than 80% FEV1/FVC ratio. Restrictive disorders are associated with decreased lung volumes (such as total lung capacity). Both restrictive and obstructive disorders are associated with a decreased functional residual capacity (for different reasons). Because lung movement is restricted, the lung compliance (ability to expand the lung) is reduced.

Which of the following is used in the treatment of acetaminophen toxicity? a. flumazenil b. naloxone C. N-acetylcysteine d. pralidoxime e. physostigmine

Choice C (N-acetylcysteine) is the correct answer. N-acetylcysteine is the antidote in cases of acetaminophen toxicity. Choice A (flumazenil) is the antidote to benzodiazepine toxicity. Choice B (naloxone) is the antidote for narcotics/opioid toxicity. Choice D (Pralidoxime) is used in the management of organophosphate and acetylcholinesterase toxicity. Choice E (Physostigmine) is an acetylcholinesterase inhibitor.

A 43-year old female presents with flank pain and hematuria for the last 48 hours. A urinalysis is performed and shows the following: Urine pH 8.2 (4.5 -8) WBCs/hpf) 15 (52-5) Leukocyte esterase Positive (Negative) Protein None (None) Nitrates Positive (Negative) RBCs/hpf 8 (</= 3) A CT scan of the abdomen and pelvis without contrast shows the presence of staghorn calculi in the right kidney. A 1-mm stone is passed and is found to be composed of ammonium magnesium phosphate (struvite). Which of the following organisms is the most likely causative agent in the formation of these stones? a. Staphylococcus saprophyticus b. Escherichia coli c. Proteus mirabilis d. Enterococcus species e. Enterobacter species

Choice C (Proteus mirabilis) is the correct answer. Staghorn calculi & struvite stones are composed of ammonium magnesium phosphate due to urea splitting organisms, which is most commonly associated with Proteus mirabilis. The splitting of urea (with ammonium as a byproduct) causes an increase in the urine pH >7. Other urea splitting organismis include: Klebsiella pneumonia and Pseudomonas aeruginosa. All the other organisms in this vignette do not produce the urease enzyme needed to split urea.

A 24-year-old woman presents with a 2-month history of leg pain when she walks more than 5 blocks. The pain is relieved shortly after rest. On physical examination, there are diminished pulses only affecting the left extremity. Blood pressure measurements between both arms are asymmetrical. An angiogram is performed and shows evidence of an aortic aneurysm. Which of the following is the most likely diagnosis? a. peripheral arterial disease b. Raynaud's phenomenon C. Takayasu arteritis d. Kawasaki disease e. Prinzmetal's angina

Choice C (Takayasu arteritis) is the correct answer. Takayasu arteritis is inflammation of the aorta, aortic arch and/or the pulmonary arteries. It most commonly presenting in women age 20-40y (especially Asian). The patients classically develop asymmetric blood pressure measurements (this finding is also classically associated with coarctation of the aorta and aortic dissection) Choice A (peripheral arterial disease) primarily involves the arteries of the lower extremities. Choice B (Raynaud's phenomenon) is spasms of the peripheral end vessels in response to cold & emotional stress. Choice D (Kawasaki disease) is seen in young children. It presents with fever and a variety of manifestations. Choice E (Prinzmetal's angina) is vasospastic disorder of the coronary arteries.

A 34-year-old female with a history of myasthenia gravis presents with respiratory muscle weakness and difficulty breathing that is worse towards the end of the day. On physical examination, lungs are clear to auscultation bilaterally and there are no crackles heard. Edrophonium is given, which worsens the symptoms including sweating, diarrhea and miosis. Which of the following is the most likely diagnosis? a. Lambert-Eaton myasthenic syndrome b. Myasthenic crisis C. Cholinergic crisis d. Adult respiratory distress syndrome e. Congestive heart failure

Choice C (cholinergic crisis) is the correct answer. Cholinergic crisis is a condition of excess acetylcholine (in this case due to acetylcholinesterase therapy). Remember that muscarinic acetylcholine receptors cause "SLUDD-C" = Salivation, Lacrimation, Urination, Defecation and Digestion as well as constriction of the pupil. Signs and symptoms include acetylcholine excess (nausea, vomiting, salivation, diarrhea, miosis, bradycardia and respiratory failure). The administration of a short-acting acetylcholinesterase inhibitor, such as Edrophonium, will aggravate the symptoms of cholinergic crisis. Choice A (Lambert-Eaton myasthenic syndrome) is a condition seen with small cell lung cancer. Patients develop weakness that improves with repeated use. It involves inhibition of presynaptic acetylcholine release. Choice B (Myasthenic crisis) is a severe presentation of myasthenia gravis with respiratory failure. A short-acting acetyl cholinesterase inhibitor such as Edrophonium will improve the symptoms of myasthenic crisis. Choice D (adult respiratory distress syndrome) and Choice E (congestive

Which of the following is required in the diagnostic evaluation of a patient suspected of having osteoporosis? a. plain radiographs of the spine and the hip b. bone scan C. dual energy x-ray absorptiometry scan d. alkaline phosphatase levels e. magnetic resonance imaging of the spine and the hip

Choice C (dual energy x-ray absorptiometry scan) is the correct answer. A DEXA scan is the diagnostic test of choice for suspected osteoporosis (defined as a T score of 5 -2.5). All of the other choices may be used as adjuncts for osteoporosis or other suspected bone disorders but is not the diagnostic test of choice.

A 67-year-old male presents with weight loss, back and bone pain. Prostate specific antigen is 20 ng/mL (normal <4 ng/mL). Which of the following would be the most likely rectal examination finding? a. boggy nontender prostate gland b. smooth firm prostate c. hard nodular prostate d. enlarged symmetric firm prostate e. tender boggy prostate

Choice C (hard nodular prostate) is the correct answer. Prostate cancer most commonly metastasizes to the bone, so an elevated PSA with weight loss and bone pain may be highly suggestive of metastatic prostate cancer. The classic physical exam finding in prostate cancer is a hard, nodular prostate gland on digital rectal examination. Choice A (boggy nontender prostate gland) is suggestive of chronic prostatitis. Choice B (smooth firm prostate) describes a normal prostate gland. Choice D (enlarged symmetric firm prostate) describes benign prostatic hypertrophy. BPH is commonly associated with elevations of prostate specific antigen. It would more likely present with obstructive or irritative symptoms but not weight loss or back pain. Choice E (tender boggy prostate) describes acute prostatitis.

A 45-year-old male with acquired immune deficiency syndrome is noncompliant with his antiretroviral therapy. He develops multiple nodulopapular, violaceous lesions on the skin consistent with Kaposi sarcoma. Which of the following is the most likely etiologic agent? a. Human papilloma virus b. Herpes simplex virus c. Human herpesvirus 8 d. Epstein Barr Virus e. Cytomegalovirus

Choice C (human herpesvirus 8) is the correct answer. Kaposi sarcoma is caused by infection of human herpes virus 8 and can be seen in patient with HIV and other immunosuppressive disorders. It is classically described as nodulopapular, violaceous lesions on the skin, gums and other areas.

Which of the following is not a side effect of lisinopril? a. teratogenicity b. hypotension c. hypokalemia d. azotemia e. dry cough and angioedema

Choice C (hypokalemia) is the correct answer. Lisinopril is an angiotensin converting enzyme inhibitor. Side effects of ACE inhibitors include teratogenicity & hypotension (it is used as an anti-hypertensive). Although ACE inhibitors are "renoprotective", they may cause azotemia in some patients with baseline kidney disease. Dry cough and angioedema are common side effects of ACE inhibition (due to an increase in bradykinin, a potent vasodilator). The inhibition of aldosterone leads to hyperkalemia.

A 43-year old female with a history of Ehlers-Danlos syndrome presents with a sudden onset of a thunderclap headache. A CT shows evidence of a subarachnoid hemorrhage. The patient is admitted to the ICU and is given phenytoin seizure prophylaxis along with bed rest. Which of the following electrolyte abnormalities are most likely to occur in this patient? a. hyperkalemia b. hypocalcemia c. hyponatremia d. hypokalemia . hypernatremia

Choice C (hyponatremia) is the correct answer. Ehlers-Danlos syndrome is a connective tissue disorder that causes fragile connective tissue, which puts patients at increased risk for developing aneurysms. Berry aneurysms are small aneurysms that occur at the circle of Willis, making patients prone to developing subarachnoid hemorrhages. Syndrome of inappropriate ADH is often caused by CNS lesions (such as subarachnoid hemorrhage) as well as anticonvulsants (such as phenytoin, which this patient has been given here). The increased ADH causes increased free water retention, which lowers the serum sodium level. Remember that hyponatremia and hyponatremia are due to water problems not problem with sodium.

A 26-year-old female presents to the clinic with intermittent episodes of diarrhea and abdominal pain that is relieved with defecation. She has occasional episodes of constipation changes in frequency. There is no evidence of blood or mucous in the stools. Which of the following is the most likely diagnosis?, a. diverticulitis b. diverticulosis c. irritable bowel syndrome d. chronic mesenteric ischemia e. volvulus

Choice C (irritable bowel syndrome) is the correct answer. Irritable bowel syndrome is classically associated with the diarrhea, constipation and abdominal pain that is classically relieved with defecation. Choice A (ischemic colitis) usually presents with abdominal pain and bloody stools (due to bowel ischemia). Choice B (diverticulosis) is the most common cause of lower gastrointestinal bleeding so it usually associated with bleeding. Choice D (chronic mesenteric ischemia) usually presents with abdominal pain that is worse with eating. Chronic mesenteric ischemia is due to atherosclerosis of the arteries that supply the bowel. It can present with "intestinal angina" - abdominal pain worse with eating (due to the increased demand of blood to the bowel in the setting of a decreased supply). Patients frequently develop anorexia due the fear of the abdominal pain associated with eating. Choice E (ulcerative colitis) often presents with left lower quadrant pain and bloody (or mucus-filled) stools.

A 20-year-old thin female presents to the clinic with multiple episodes of chest pain, panic attacks, and dizziness whenever she exercises or exerts herself. On physical examination, there is the presence of a systolic ejection click. Which of the following is the recommended management of this patient? a. reassurance that the disease is self limiting b. education that she may develop the murmur of mitral stenosis c. propranolol for the autonomic symptoms d. aspirin to reduce the chest pain symptoms e. nitroglycerin as needed for the chest pain and other symptoms

Choice C (propranolol for the autonomic symptoms) is the correct answer. In patients with mitral valve prolapse with autonomic symptoms, beta blockers can be given to reduce the symptoms. Otherwise, in the majority of patients with mitral valve prolapse without significant autonomic symptoms, observation and reassurance is the treatment of choice ( Choice A). This is a key point to remember. Without significant autonomic symptoms, choose reassurance. Choice B (education that she may develop the murmur of mitral stenosis) is incorrect as mitral valve prolapse is not associated with mitral stenosis. Choice D (aspirin to reduce the chest pain symptoms) and Choice E (nitroglycerin as needed for the chest pain and other symptoms) are not indicated in the autonomic symptoms associated with mitral valve prolapse in some patients.

A 43-year-old patient is diagnosed with active tuberculosis. Ethambutol is one of the four drugs used to treat his active disease. A week into the treatment, he develops painless central blind spots in the right eye. On fundoscopic examination, there is a normal disc to cup ratio. During a swinging light test, when the light is shone from the left eye to the right eye, the pupil appears to dilate. The cornea looks normal in appearance and there is no conjunctival erythema. There is no blurring of the optic disc on fundoscopic examination. Which of the following is the most likely diagnosis? a. papilledema b. papillitis C. retrobulbar neuritis d. acute angle closure glaucoma e. chronic open angle glaucoma

Choice C (retrobulbar neuritis) is correct choice. Ethambutol is a drug used to treat active tuberculosis and can cause optic neuritis (inflammation of the optic nerve). There are two types of optic neuritis, retrobulbar (optic nerve inflammation behind the eye) and papillitis (edema of the optic nerve at the eye). In this vignette, there is no blurring of the optic disc on fundoscopic exam, indicating retrobulbar neuritis. If there was evidence of blurring of the optic disc, then choice B (papillitis) would have been the correct answer. EENT - OPTIC NEURITIS - MOST LIKELY (p 227). Choice A (papilledema) is associated with blurring of the optic disc due to increased cerebrospinal fluid pressure, as seen with benign intracranial hypertension or malignant hypertension. Choice D (acute angle closure glaucoma) is classically associated with blurring of the optic disc due to increased intraocular pressure. Medications that can promote the development of acute angle closure glaucoma are anticholinergics and sympathomimetics (because these medications cause dilation of the pupils which can close off the pre-existing narrow angle). Ethambutol is associated with optic neuritis. Choice E (chronic open angle closure glaucoma) can cause blurring of the optic disc as well and is usually bilateral.

Which of the following classically describes the appearance of seborrheic keratosis? a. Red elevated nodule with adherent white scales with crusted blood at the margins b. dry, rough scaly, "sandpaper" like rash with hyperkeratotic plaques c. small papule or plaque with a velvety, warty, "stuck on" appearance d. small, raised translucent pearly papule with central ulceration and rolled borders. e. Asymmetric, multi-color, 7-mm lesion with irregular borders and rapid change in appearance

Choice C (small papule or plaque with a velvety, warty, "stuck on" appearance) is the correct answer. This is the classic description of seborrheic keratosis, a benign skin tumor most commonly seen in fair skinned elderly patients with prolonged sun exposure. Choice A (red elevated nodule with adherent white scales and crusted blood at the margins) describes squamous cell carcinoma. Choice B (dry, rough scaly, "sandpaper" like rash with hyperkeratotic plaques) describes actinic keratosis, a premalignant condition that may lead to squamous cell carcinoma. Choice D (small, raised translucent pearly papule with central ulceration and rolled borders) is the classic description of basal cell carcinoma. Choice E (asymmetric, multi-color, 7-mm lesion with irregular borders and rapid change in appearance) describes malignant melanoma.

Which of the following is not a side effect of long-term lithium therapy? a. hyperparathyroidism b. hypothyroidism c. syndrome of inappropriate antidiuretic hormone secretion d. arrhythmias e. seizures

Choice C (syndrome of inappropriate antidiuretic hormone) is the correct answer. Lithium causes diabetes insipidus. Other side effects of lithium therapy include: increased thirst, seizures, arrhythmias, hypercalcemia, hyperparathyroidism, and hypothyroidism.

In a patient with normal blood urea nitrogen and creatinine levels, which of the following radiograph findings are most consistent with osteomalacia not due to renal disease? a. "salt and pepper" appearance of the skull b. "punched out lesions" appearance of the skull c. the presence of Looser zones d. "cotton wool" appearance of the skull e. pathologic fractures and radiologic evidence of kyphosis

Choice C (the presence of Looser zones) is the correct answer. Osteomalacia is due to vitamin D deficiency in adults. It is associated with demineralization of the bone. In some areas, the demineralized bone (osteoid) can give the appearance of a fracture. This is called a Looser line or Looser zone. Choice A ("salt and pepper" appearance of the skull) is associated with renal osteodystrophy, a special type of osteomalacia seen in chronic renal disease. The secondary hyperparathyroidism associated with decreased vitamin D production causes increased bone resorption in the skull, leading to the salt and pepper appearance. Choice B ("punched out lesions" appearance of the skull) is associated with multiple myeloma. Choice D ("cotton wool" appearance of the skull) is associated with Paget's disease of the bone. Choice E (pathologic fractures and radiologic evidence of kyphosis) is seen with osteoporosis.

A 32-year-old male with no past medical or psychiatric history has a 2-month history of rapid functional decline as per his family. Today, he was found on the subway platform saying that "the voices in his head kept telling him to jump off the platform." He states he was contemplating jumping in an attempt to stop the sensation of "insects crawling on his body". He is sitting on the stretcher wearing a hat made of foil paper to protect his thoughts from being intercepted by the Chinese government. He seems to have a flat effect when communicating with the health care providers. Which of the following is the most likely diagnosis? a. schizophrenia b. Bipolar disorder C. Paranoid personality disorder d. Schizophreniform disorder e. Schizoid personality disorder

Choice D (Schizophreniform disorder) is the correct choice. The patient meets the criteria for schizophrenia but the symptoms are less than 6 months in duration, making the diagnosis of Schizophreniform a more likely diagnosis. The criteria for schizophrenia are functional decline in addition to hallucinations and delusions. The main difference between the two is primarily the length of time. Choice E (Schizoid personality disorder) is characterized by long pattern of voluntary social withdrawal, hermit-like behavior and a cold affect.

A 34-year-old male with a history of multiple endocrine neoplasia type 2B is being evaluated. Which of the following would most likely be elevated if thyroid biopsy were positive for medullary thyroid carcinoma? a. thyroid stimulating hormone levels b. thyroid stimulating antibody levels C. thyroid peroxidase antibody levels d. calcitonin levels e. parathyroid hormone levels

Choice D (calcitonin levels) is the correct answer. Medullary thyroid carcinoma arises from the parafollicular c cells. The normal job of the parafollicular cells is to secrete calcitonin in response to hypocalcemia. Patients with medullary thyroid carcinoma will have elevated calcitonin levels so it can be used as a tumor marker in these patients to monitor tumor burden, effective treatment and tumor recurrence. Choice A (thyroid stimulating hormone levels) is the first test ordered for suspected thyroid disorders. Patients with thyroid malignancies are often euthyroid, so thyroid function tests are often be normal. Choice B (thyroid stimulating antibody levels) is classically associated with Graves' disease. Choice C (thyroid peroxidase antibody level) is classically associated with autoimmune thyroiditis, such as Hashimoto's, silent lymphocytic or post-partum thyroiditis. Choice E (parathyroid hormone level) is used to evaluate suspected parathyroid disorders.

Which of the following tests best differentiates nephrogenic diabetes insipidus from central diabetes insipidus? a. fluid challenge test b. dexamethasone suppression test C. fluid deprivation test d. desmopressin stimulation test e. Captopril test

Choice D (desmopressin stimulation test) is the correct answer. Central diabetes insipidus is the absence of ADH, Nephrogenic diabetes insipidus is kidney insensitivity to ADH. To distinguish between the two, desmopressin (ADH) is given and then urine osmolarity is measured. In central diabetes, the synthetic desmopressin will cause a gradual increase in urine osmolarity, indicating kidney responsiveness to ADH. In nephrogenie diabetes insipidus, the desmopressin is ineffective, leading to the continued production of dilute urine (indicating the kidney's unresponsiveness to ADH). Choice A (fluid challenge test) is used in patients with decreased urine output to see if there is an increased in urine output, which would indicate prerenal azotemia, Choice B (dexamethasone suppression test) is used in the evaluation of suspected Cushing's syndrome. Choice C (fluid deprivation test) is the screening test for suspected diabetes insipidus. After screening, a desmopressin test is done to distinguish central from nephrogenic diabetes insipidus. Choice E (Captopril test) is used in the evaluation of patients with suspected renal artery stenosis.

A patient has routine hepatitis B testing with the following lab values: Hepatitis B surface antigen: negative Hepatitis B surface antibody: positive Hepatitis B core antibody IgM: negative Hepatitis B core antibody IgG: positive Hepatitis E antigen: negative Which of the following is the most likely diagnosis? a. The patient has acute hepatitis B infection b. The patient is in the window period C. The patient has been successfully vaccinated d. The patient has full recovery of distant hepatitis B infection e. The patient has chronic hepatitis B infection

Choice D (distant, resolved hepatitis B infection) is the correct answer. The presence of positive surface antibodies and positive core IgG antibodies indicates a distant resolved infection. Choice A (acute hepatitis B infection) is incorrect. Acute hepatitis B infection is indicated by the presence of positive Hepatitis B surface antigen with positive core IgM antibodies. Choice B (window period) is incorrect. The only positive marker in the window period is the hepatitis B core antibody IgM. Choice C (The patient has been successfully vaccinated) is incorrect. The only marker positive after successful vaccination is a positive surface antibody. This is distinguished from distant resolved infection by the lack of the core antibody positivity in vaccinated individuals. Choice E (The patient has chronic hepatitis B infection) is incorrect. Lab wise, chronic hepatitis B infection is associated with positive surface antigen and a positive core Ig G for 6 months. Hepatitis E is indicative of active replication.

Which of the following is classically seen in osteoarthritis? a. warm, boggy, and edematous joint b. positive rheumatoid factor, elevated C-reactive protein and erythrocyte sedimentation rate c. symmetric joint narrowing d. evening stiffness predominantly and morning stiffness lasting less than 30 minutes e. osteopenia

Choice D (evening stiffness predominantly and morning stiffness lasting less than 30 minutes) is the correct answer. All of the other choices are classically associated with rheumatoid arthritis.

A 53-year-old male with a history of adult polycystic kidney disease presents to the emergency room with a sudden onset of "the worse headache of my life" that was followed by a witnessed brief loss of consciousness. The patient is now experiencing headache, nausea, vomiting and nuchal rigidity. There are no focal neurological deficits and he is afebrile. There is no papilledema present on fundoscopic examination. A CT scan of the head shows no evidence of hemorrhage or mass lesion. A lumbar puncture is performed. Which of the following cerebrospinal fluid (CSF) findings would be most likely present? a. increased CSF opening pressure with an otherwise normal CSF examination b. high protein with normal white blood cell count c. increased oligoclonal IgG bands d. increased CSF opening pressure with xanthochromia that does not diminish from tube 1 to tube 4 e. normal glucose with increased white blood cells (primarily lymphocytes)

Choice D (increased CSF opening pressure xanthochromia that does not diminish from tube 1 to tube 4) is the correct answer. The neurological symptoms, nuchal rigidity, sudden onset of the "worse of headache of my life", the loss of consciousness with a lucid interval followed by decompensation are hallmark for subarachnoid hemorrhage. Extrarenal manifestations of polycystic kidney disease include the presence of mitral valve prolapse and an increased propensity towards the development of berry aneurysms, which may rupture causing a subarachnoid hemorrhage. Choice A (increased CSF opening pressure with an otherwise normal CSF examination) would be seen with pseudo tumor cerebri/idiopathic intracranial hypertension. Pseudotumor cerebri usually presents with headache and may often develop visual changes. It is not associated with loss of consciousness. Choice B (high protein with normal white blood cell count) is the characteristic CSF finding in patients with Guillain Barré syndrome. Guillain Barré classically presents with symmetric lower extremity weakness that progresses to involve the upper extremities. Choice C (increased oligoclonal IgG bands) is the classic CSF finding in patients with multiple sclerosis. Choice E [normal glucose with increased white blood cells (primarily lymphocytes)] is the classic CSF finding in viral meningitis, aseptic meningitis and encephalitis.

A 45-year-old male presents with non-bloody diarrhea and crampy abdominal pain (especially in the right lower quadrant). Rovsing, Obturator and Psoas signs are negative. He undergoes an upper GI series with small bowel follow through and a "string sign" is seen in the terminal ileum. The patient is saccharomyces cerevisiae antibody positive. Which of the following lab findings would most likely be seen in this patient? a. increased alpha fetoprotein b. increased perinuclear anti-neutrophil cytoplasmic antibodies c. increased smooth muscle antibodies d. increased mean corpuscular volume of the red blood cells e. increased endomysial antibodies

Choice D (increased MCV) is the correct answer. The string sign, non-bloody diarrhea and right lower quadrant pain is highly suggestive of Crohn's disease. The terminal ileum is the most common site of Crohn's disease and since it is located in the right lower quadrant, RLQ pain is a common finding in patients with Crohn's. Both inflammatory bowel diseases (Crohn's and Ulcerative Colitis) are thought to arise from an inappropriate autoimmune response to the normal GI flora. Antibodies against the harmless saccharomyces cerevisiae (Baker's/Brewer's yeast) is seen in about 70% of patients with Crohn's (and only 10-15% of those with ulcerative colitis so that also leads to the diagnosis of Crohn's). Because Crohn's most commonly affects the terminal ileum (where B12 is absorbed), patients may develop a B12 deficiency and a subsequent macrocytic anemia with an increased mean corpuscular volume (MCV) of the red blood cell.

A 44-year-old woman is complaining of decreased vision and pain in the right eye especially when staring at lights. On physical examination, there is conjunctival erythema with the presence of a ciliary flush. A slit lamp reveals the presence of dendritic lesions. Which of the following is the most appropriate therapy? a. pilocarpine ophthalmic solution b. olopatadine ophthalmic solution c. prednisolone ophthalmic solution d. trifluridine ophthalmic drops and acyclovir orally e. polymyxin B/trimethoprim ophthalmic solution

Choice D (trifluridine ophthalmic drops and acyclovir orally) is the correct choice. The presence of dendritic lesions in this vignette indicates herpes ophthalmic keratitis. Keratitis is associated with visual changes and the presence of a limbic (ciliary) flush. Treatment for herpes keratitis includes the treatment with antiviral medications Choice A (pilocarpine drops) is used in acute angle closure glaucoma or to treat Sjrogen's syndrome. Pilocarpine is a cholinergic drug, which leads to pupillary constriction (reducing the angle closure in glaucoma). Pilocarpine causes increased tear and saliva production (treating the dry eye & mouth seen in Sjrogen's syndrome) Choice B (olopatadine ophthalmic) is an antihistamine drop used to treat inflammation or chemosis in patients with allergic or viral conjunctivitis.8yh Choice C (prednisolone ophthalmic) can be used in patients with anterior uveitis. Which can also present with limbic (ciliary flush) as well as cells and flare on ophthalmic exam but would not be associated with dendritic lesions. Choice E (polymyxin B/trimethoprim ophthalmic solution) is used in the management of conjunctivitis.

A 33-year-old male has been complaining of progressive shortness of breath with exertion. Physical examination reveals pursed lip breathing, increased anteroposterior diameter, decreased fremitus, hyperresonance to percussion and decreased breath sounds. Chest radiographs show darkened lung fields and flattened diaphragms. Upon questioning the patient, he says he has never smoked nor lived with smokers. Which of the following tests would most likely show the etiology of his symptoms? a. angiotensin converting enzyme levels b. Ig G antibodies to the type IV collagen of the alveoli c. anti-double stranded DNA antibodies d. sweat chloride test e. alpha-1 antitrypsin levels

Choice E (Alpha antitrypsin) is the correct choice. Suspect AAT deficiency in a young patient with emphysema who is not a smoker. Choice A (angiotensin converting enzyme) is elevated in sarcoidosis (which is restrictive and associated with decreased lung volumes not increased lung volumes seen in this vignette). Choice B (IgG in alveoli) is hallmark of Goodpasture's syndrome, causing hemoptysis & rapid kidney failure. Choice C (anti-double stranded DNA antibodies) are seen with lupus. Choice D (sweat chloride test) is used to diagnose cystic fibrosis.

A 45-year-old female seeks medical therapy for her recurrent symptoms that occur multiple times over the last year. She has had a longstanding history of painful legs, difficulty swallowing, vomiting, intermittent dyspnea and dysmenorrhea. She has had multiple workups, of which no organic cause was found for her varied symptoms. She is convinced that surgery may be curative since "medicine doesn't seem to help". Which of the following is the most likely diagnosis? a. Body dysmorphic disorder b. Functional neurological disorder (Conversion disorder) C. Malingering d. Factitious disorder (Munchausen syndrome) e. Somatic symptom disorder (Somatization disorder)

Choice E (Somatization disorder) is correct. Somatic symptom disorder (somatization disorder) is characterized by chronic physical symptoms involving more than 1 body part with no physical cause of the symptoms. Choice A (body dysmorphic disorder) is characterized by an excessive preoccupation that a body part (or slight anomaly) is deformed, which often causes them to be ashamed. Choice B (functional neurological disorder/conversion disorder) is a loss of motor or sensory neurologic function suggestive of a physical disorder but caused by psychological Choice C (malingering) is fabricating or exaggerating the symptoms of physical or mental disorders for a multitude of "secondary gains" (ex. financial compensation, avoiding work etc.). Choice D (Munchausen syndrome) is an intentional, self-induced symptom or falsified physical or lab findings. This can include the seeking of multiple invasive procedures and operations even if they pose a serious life risk.

"H-shaped" vertebrae with central endplate depressions are seen with which of the following disorders? a. Spondylolisthesis b. Spondylolysis C. Ankylosing spondylitis d. Spinal stenosis e. Sickle cell disease

Choice E (sickle cell disease) is the correct answer. This is a common radiograph finding due to micro infarctions leading to central endplate depression of the vertebrae. Choice A (Spondylolisthesis) shows up on radiographs as forward slipping of the vertebrae on one another. Choice B (Spondylolysis) is a defect seen in the pars interarticularis. Choice C (Ankylosing spondylitis) is associated with a bamboo spine on radiographs. Choice D (Spinal stenosis) is associated with narrowing of the spinal canal.

A 35-year-old male comes to the clinic for a routine physical examination. During obtaining a history, he tells you his brother was diagnosed with colon cancer at 53-vears-old. Which of the following describes the appropriate colon cancer screening guidelines for this patient? a. Fecal occult blood testing now and colonoscopy every 10 years b. Fecal occult blood testing at age 40 and colonoscopy every 10 years C. Fecal occult blood testing at age 50 and colonoscopy every 10 years d. Fecal occult blood testing at age 50 and colonoscopy every 5 years e. Fecal occult blood testing at age 40 and colonoscopy every 5 years

Choice E is correct. A first-degree relative is a parent, sibling or offspring. Because his brother was diagnosed at age 53, recommended age is to begin at age 40 (or 10 years before the person was diagnosed which would have been 43) so you use the lower number. The 10 year period comes from the thought it takes roughly about 10 years on average for a polyp to become malignant. Choice B (Fecal occult blood testing at age 40 and colonoscopy every 10 years) is recommended in patients whose first degree relative was diagnosed with colon cancer over the age of 60. Choice C (Fecal occult blood testing at age 50 and colonoscopy every 10 years)

Which of the following is a potential adverse effect associated w/ unfractionated low molecular weight heparin? a. excessive cough b. hyperglycemia c. hypothyroidism d. muscle cramps e. thrombocytopenia

E. Heparin-induced thrombocytopenia (HIT) is a potentially serious complication of unfractionated without heparin therapy and has been reported in up to 5% of patients exposed to heparin for more than 4 days.

Which of the following most reliably distinguishes chronic primary adrenocortical insufficiency from secondary adrenocortical insufficiency? a. the presence of fatigue b. the presence of hyperkalemia c. the presence of muscle weakness d. the presence of hyponatremia e. the presence of skin hyperpigmentation

Primary adrenal insufficiency is lack of the adrenal cortex production of cortisol, aldosterone (and androgen in females). The low levels of cortisol stimulate increased pituitary ACTH secretion. The increased ACTH levels causes increases in melanocyte stimulating hormone levels (both ATCH & MSH made from the same precursor molecule), leading to hyperpigmentation. In secondary, there is decreased ACTH production, so there is no hyperpigmentation. All of the other symptoms can be seen in both primary and secondary adrenal insufficiency.

What type of cardiomyopathy is caused by amyloidosis?

Restrictive cardiomyopathy Presents w/ peripheral edema, dyspnea, fatigue, (signs of heart failure) Dx-echo will show impaired diastolic filling and preserved systolic function

Which of the following has the potential for causing cyanide toxicity? a. clonidine b. diazoxide c. reserpine d.hydralazine e. sodium nitroprusside

e. sodium nitroprusside Intravenous nitroprusside is a medication used for hypertensive emergencies. It is metabolized to cyanide and then to thiocyanate, which is excreted in the urine. Risk factors for nitroprusside-induced cyanide poisoning include a prolonged treatment period (>24-48 hours), underlying renal impairment, and the use of doses that exceed the capacity of the body to detoxify cyanide (i.e., more than 2 μg/kg per minute).


Ensembles d'études connexes

SFTY 6006 - Chapter 2 - Legislative Framework

View Set

First Amendment: Freedom of Expression

View Set

vSim Health Assessment | Jared Griffin (Cardiovascular Assessment)

View Set